Art of Problem Solving

2025 AMC 12A Problems/Problem 4: Difference between revisions

Daniel.ariew (talk | contribs)
Created page with "{{duplicate|2025 AMC 10A #8 and 2025 AMC 12A #4}}"
 
Tfea (talk | contribs)
 
(13 intermediate revisions by 7 users not shown)
Line 1: Line 1:
{{duplicate|[[2025 AMC 10A Problems/Problem 8|2025 AMC 10A #8]] and [[2025 AMC 12A Problems/Problem 4|2025 AMC 12A #4]]}}
{{duplicate|[[2025 AMC 10A Problems/Problem 8|2025 AMC 10A #8]] and [[2025 AMC 12A Problems/Problem 4|2025 AMC 12A #4]]}}
==Problem==
Agnes writes the following four statements on a blank piece of paper.
<imath>\bullet</imath> At least one of these statements is true.
<imath>\bullet</imath> At least two of these statements are true.
<imath>\bullet</imath> At least two of these statements are false.
<imath>\bullet</imath> At least one of these statements is false.
Each statement is either true or false. How many false statements did Agnes write on the paper?
<imath>\textbf{(A) } 0 \qquad\textbf{(B) } 1 \qquad\textbf{(C) } 2 \qquad\textbf{(D) } 3 \qquad\textbf{(E) } 4</imath>
==Solution 1==
We first number all the statements:
1) At least one of these statements is true.
2) At least two of these statements are true.
3) At least two of these statements are false.
4) At least one of these statements is false.
We can immediately see that statement 4 must be true, as it would contradict itself if it were false. Similarly, statement 1 must be true, as all the other statements must be false if it were false, which is contradictory because statement 4 is true. Since both 1 and 4 are true, statement 2 has to be true. Therefore, statement 3 is the only false statement, making the answer <imath>\boxed{\text{(B) }1}</imath>.
-Rainjs
==Solution 2==
Statements <imath>I,II,</imath> and <imath>IV</imath> are true, while statement <imath>III</imath> is false. Hence, there are <imath>3</imath> true statements and <imath>\boxed{\text{(B) }1}</imath> false statement. This result can be checked by examining the statements individually again.
Statements <imath>I</imath> and <imath>II</imath> will be true because there are <imath>3\ge2</imath> true statements. Statement <imath>IV</imath> is also true because there is <imath>1\ge1</imath> false statement. Finally, statement <imath>III</imath> is false because there are <imath>1\ngeq2</imath> false statements.
~Tacos_are_yummy_1
==Solution 3==
Let's say there are <imath>T</imath> true statements. We know that <imath>T</imath> can be any integer from <imath>0</imath> to <imath>4</imath>. We denote <imath>A</imath> as <imath>T \geq 1</imath>, Statement <imath>B</imath> as <imath>T \geq 2</imath>, Statement <imath>C</imath> as <imath>T \leq 2</imath>, and Statement <imath>D</imath> as <imath>T \leq 3</imath>.
If <imath>T=0</imath>, then <imath>C</imath> and <imath>D</imath> are met, so there are <imath>2</imath> true statements, which is a contradiction.
If <imath>T=1</imath>, then <imath>A,C,D</imath> are met, so there are <imath>3</imath> true statements, which is a contradiction.
If <imath>T=2</imath>, then <imath>A,B,C,D</imath> are met, so there are <imath>4</imath> true statements, which is a contradiction.
If <imath>T=3</imath>, then <imath>A,B,D</imath> are met, so there are <imath>3</imath> true statements, which is consistent with our assumption that <imath>T=3</imath>.
If <imath>T=4</imath>, then <imath>A,B</imath> are met, so there are <imath>2</imath> true statements, which is a contradiction.
Only <imath>T=3</imath> was consistent, so there are <imath>3</imath> true statements and <imath>4-3=\boxed{1}</imath> false statement. (In particular, Statement C is the false statement).
~lprado
==Solution 4==
Suppose Statement <imath>I</imath> is false, then none of the Statements are true, which contradicts the fact that a false Statement <imath>III</imath> or <imath>IV</imath> is telling the truth. Therefore, Statement <imath>I</imath> is true and assume Statement <imath>II</imath> is false.
Statement <imath>II</imath> thus implies that only Statement <imath>I</imath> was the truth, and the rest, false. But then, there are 3 false statements but then Statement <imath>III</imath> and Statement <imath>IV</imath> are telling the truth. So Statement <imath>II</imath> is also true.
Now, if Statement <imath>III</imath> is true, then both Statement <imath>III</imath> and Statement <imath>IV</imath> is false, contradicting the fact that it is true. Statement <imath>III</imath> is hence false and Statement <imath>IV</imath> tells the truth since Statement <imath>III</imath> lied so indeed, there are at least one lie. There are a total of 3 truths and 1 lie, making the answer <imath>\boxed{\text{(B) }1}</imath>. ~hxve
==Solution 5 (Quick)==
I tried setting that the statements regarding the true ones (I and II) were true and
regarding the false ones (III and IV) were false, and it clearly doesn't work.
It doesn't work because statement III was true, not false as I set it to be originally.
What if we make that statement (Statement III) true too?
Well, then we have 3 T and 1 F statements, so statement IV is false,
statement III is true, and statements I and II are true too.
So there is <imath>\boxed{\text{(B) }1}</imath> false statement.
~Aarav22
==Solution 6 (Plug in Answer Choices)==
For this question, what I did was to simply test each of the answer choices.
Let's start with (A) 0 false statements:
We can check the answer of 0 false statements against each of the statements.
Statement I: "At least one of these statements is true."
This statement is false, which already invalidates the answer of (A) 0 false statements, as we already have a false statement.
Let's move onto (B) 1 false statement:
We can check once again this answer against each of the statements.
Statement I: "At least one of these statements is true."
This statement is true, as when we have 1 false statement, we have 3 true statements, and therefore this is true.
Statement II: "At least two of these statements are true."
Once again, this statement is true, as when we have 1 false statement, we have 3 true statements, and thus this is also true.
Statement III: "At least two of these statements are false."
This statement is false, as we have one false statement, not 2 and above.
Statement IV: "At least one of these statements is false."
This statement is true, as we have one false statement.
So, after checking the answer choice of (B) 1 false statement across the 4 statements, we get three true statements and one false statement, making this statement the correct answer. I will not dive into the other 3 answer choices, as we have already gotten the answer for this question, and if you were doing this way of solving on the test, you would most likely not test the other 3 answers either for the sake of saving time.
~EZ123 (Would appreciate it if someone formatted my solution using latex, thanks!)
==Solution 7 (Step by Step,Simple)==
Start by going statement by statement and checking if they can be false.If statement 1 were false statement 4 would be true leading to a contradiction so statement 1 is true.If statement 2 were false 4 would be true and one would be true making statement 2 true.If statement 3 was false and we know statement 1 and statement 2 were true and 4 can be true and it works.Therefore the correct answer is B)1.
--TFEA(would also appreciate if somebody could please latex my proof).
==Video Solution by Power Solve==
https://youtu.be/QBn439idcPo?si=DGqtuDIJ399xE_rh&t=524
==Chinese Video Solution==
https://www.bilibili.com/video/BV1t72uBREof/
~metrixgo
==Video Solution by SpreadTheMathLove==
https://www.youtube.com/watch?v=dAeyV60Hu5c
== Video Solution (In 1 Min) ==
https://youtu.be/uv3uIMwIkrg?si=XCbsXL7ikMawCGyM ~ Pi Academy
==Video Solution by Daily Dose of Math==
https://youtu.be/gPh9w3X3QSw
~Thesmartgreekmathdude
==See Also==
{{AMC10 box|year=2025|ab=A|num-b=7|num-a=9}}
{{AMC12 box|year=2025|ab=A|num-b=3|num-a=5}}
{{MAA Notice}}

Latest revision as of 14:56, 11 November 2025

The following problem is from both the 2025 AMC 10A #8 and 2025 AMC 12A #4, so both problems redirect to this page.

Problem

Agnes writes the following four statements on a blank piece of paper.

$\bullet$ At least one of these statements is true.

$\bullet$ At least two of these statements are true.

$\bullet$ At least two of these statements are false.

$\bullet$ At least one of these statements is false.

Each statement is either true or false. How many false statements did Agnes write on the paper?

$\textbf{(A) } 0 \qquad\textbf{(B) } 1 \qquad\textbf{(C) } 2 \qquad\textbf{(D) } 3 \qquad\textbf{(E) } 4$

Solution 1

We first number all the statements:

1) At least one of these statements is true. 2) At least two of these statements are true. 3) At least two of these statements are false. 4) At least one of these statements is false.

We can immediately see that statement 4 must be true, as it would contradict itself if it were false. Similarly, statement 1 must be true, as all the other statements must be false if it were false, which is contradictory because statement 4 is true. Since both 1 and 4 are true, statement 2 has to be true. Therefore, statement 3 is the only false statement, making the answer $\boxed{\text{(B) }1}$. -Rainjs

Solution 2

Statements $I,II,$ and $IV$ are true, while statement $III$ is false. Hence, there are $3$ true statements and $\boxed{\text{(B) }1}$ false statement. This result can be checked by examining the statements individually again.

Statements $I$ and $II$ will be true because there are $3\ge2$ true statements. Statement $IV$ is also true because there is $1\ge1$ false statement. Finally, statement $III$ is false because there are $1\ngeq2$ false statements.

~Tacos_are_yummy_1

Solution 3

Let's say there are $T$ true statements. We know that $T$ can be any integer from $0$ to $4$. We denote $A$ as $T \geq 1$, Statement $B$ as $T \geq 2$, Statement $C$ as $T \leq 2$, and Statement $D$ as $T \leq 3$.

If $T=0$, then $C$ and $D$ are met, so there are $2$ true statements, which is a contradiction.

If $T=1$, then $A,C,D$ are met, so there are $3$ true statements, which is a contradiction.

If $T=2$, then $A,B,C,D$ are met, so there are $4$ true statements, which is a contradiction.

If $T=3$, then $A,B,D$ are met, so there are $3$ true statements, which is consistent with our assumption that $T=3$.

If $T=4$, then $A,B$ are met, so there are $2$ true statements, which is a contradiction.

Only $T=3$ was consistent, so there are $3$ true statements and $4-3=\boxed{1}$ false statement. (In particular, Statement C is the false statement).

~lprado

Solution 4

Suppose Statement $I$ is false, then none of the Statements are true, which contradicts the fact that a false Statement $III$ or $IV$ is telling the truth. Therefore, Statement $I$ is true and assume Statement $II$ is false.

Statement $II$ thus implies that only Statement $I$ was the truth, and the rest, false. But then, there are 3 false statements but then Statement $III$ and Statement $IV$ are telling the truth. So Statement $II$ is also true.

Now, if Statement $III$ is true, then both Statement $III$ and Statement $IV$ is false, contradicting the fact that it is true. Statement $III$ is hence false and Statement $IV$ tells the truth since Statement $III$ lied so indeed, there are at least one lie. There are a total of 3 truths and 1 lie, making the answer $\boxed{\text{(B) }1}$. ~hxve

Solution 5 (Quick)

I tried setting that the statements regarding the true ones (I and II) were true and regarding the false ones (III and IV) were false, and it clearly doesn't work. It doesn't work because statement III was true, not false as I set it to be originally.

What if we make that statement (Statement III) true too? Well, then we have 3 T and 1 F statements, so statement IV is false, statement III is true, and statements I and II are true too.

So there is $\boxed{\text{(B) }1}$ false statement.

~Aarav22

Solution 6 (Plug in Answer Choices)

For this question, what I did was to simply test each of the answer choices.

Let's start with (A) 0 false statements:

We can check the answer of 0 false statements against each of the statements.

Statement I: "At least one of these statements is true."

This statement is false, which already invalidates the answer of (A) 0 false statements, as we already have a false statement.

Let's move onto (B) 1 false statement:

We can check once again this answer against each of the statements.

Statement I: "At least one of these statements is true."

This statement is true, as when we have 1 false statement, we have 3 true statements, and therefore this is true.

Statement II: "At least two of these statements are true."

Once again, this statement is true, as when we have 1 false statement, we have 3 true statements, and thus this is also true.

Statement III: "At least two of these statements are false."

This statement is false, as we have one false statement, not 2 and above.

Statement IV: "At least one of these statements is false."

This statement is true, as we have one false statement.

So, after checking the answer choice of (B) 1 false statement across the 4 statements, we get three true statements and one false statement, making this statement the correct answer. I will not dive into the other 3 answer choices, as we have already gotten the answer for this question, and if you were doing this way of solving on the test, you would most likely not test the other 3 answers either for the sake of saving time.

~EZ123 (Would appreciate it if someone formatted my solution using latex, thanks!)

Solution 7 (Step by Step,Simple)

Start by going statement by statement and checking if they can be false.If statement 1 were false statement 4 would be true leading to a contradiction so statement 1 is true.If statement 2 were false 4 would be true and one would be true making statement 2 true.If statement 3 was false and we know statement 1 and statement 2 were true and 4 can be true and it works.Therefore the correct answer is B)1.

--TFEA(would also appreciate if somebody could please latex my proof).

Video Solution by Power Solve

https://youtu.be/QBn439idcPo?si=DGqtuDIJ399xE_rh&t=524

Chinese Video Solution

https://www.bilibili.com/video/BV1t72uBREof/

~metrixgo

Video Solution by SpreadTheMathLove

https://www.youtube.com/watch?v=dAeyV60Hu5c

Video Solution (In 1 Min)

https://youtu.be/uv3uIMwIkrg?si=XCbsXL7ikMawCGyM ~ Pi Academy

Video Solution by Daily Dose of Math

https://youtu.be/gPh9w3X3QSw

~Thesmartgreekmathdude

See Also

2025 AMC 10A (ProblemsAnswer KeyResources)
Preceded by
Problem 7
Followed by
Problem 9
1 2 3 4 5 6 7 8 9 10 11 12 13 14 15 16 17 18 19 20 21 22 23 24 25
All AMC 10 Problems and Solutions
2025 AMC 12A (ProblemsAnswer KeyResources)
Preceded by
Problem 3
Followed by
Problem 5
1 2 3 4 5 6 7 8 9 10 11 12 13 14 15 16 17 18 19 20 21 22 23 24 25
All AMC 12 Problems and Solutions

These problems are copyrighted © by the Mathematical Association of America.